1
$\begingroup$

Let $W$ be a one-dimensional standard Brownian motion and denote $$X_t=-\mu t + \sigma W_t, \quad t\ge 0,$$ where $\mu$ and $\sigma$ are positive constants. For $b<0$ denote the first passage time of level $b$ by $\tau$: $$ \tau:=\inf\{t\ge 0: X_t=b\}.$$ My question is: how can one find $${\mathbb E}\left[\int_0^\tau X_t dt \right]?$$ Or slightly more general, $${\mathbb E}\left[\int_0^\tau f(X_t) dt \right]$$ for some measurable function $f$?

$\endgroup$

1 Answer 1

5
$\begingroup$

Define $u(x)=E^x\int_0^\tau X_s ds$, then $u$ satisfies $\sigma^2 u_{xx}/2-\mu u_x=-x$ with boundary condition $u(b)=0$ and $u(\infty)=\infty$. (This is missing a boundary condition, but a good way to discover the extra boundary condition at $b$ is to solve first in a strip and then take the width of the strip to infinity). Now solve the ODE (solution is explicit). When $f$ is involved, replace RHS of ODE by $f(x)$. I am not sure this is research level question.

$\endgroup$
2
  • $\begingroup$ Professor Zeitouni, thanks a lot for your answer. I solved the ODE with two free parameters: $u(x)=A \exp(2\mu x/\sigma^2)+x^2/2\mu + \sigma^2 x/2\mu^2 + B$. The boundary condition $u(b)=0$ gives me one equation for the parameters $A,B$; but the other condition $u(\infty)=\infty$ does not give me an explicit equation. In fact, for any $A\ge 0$, I can find a $B$ so that both boundary conditions are satisfied. In other words, the solution to the ODE is not unique? Maybe I am still missing something. $\endgroup$
    – epsilon
    Aug 27, 2013 at 16:33
  • $\begingroup$ As I wrote above: solve first in a bounded interval $[b,R]$ with both boundary conditions $0$. This gives you an $R$ dependent solution, and now take $R\to\infty$. This should give you the answer. (There is a technical point that indeed convergence occurs, but it is not too hard to prove, because of the fact that when starting at $x$, the probability to hit $+R$ decays exponentially as $R\to\infty$ while the gain in the value is only polynomial.) $\endgroup$ Aug 27, 2013 at 18:45

Your Answer

By clicking “Post Your Answer”, you agree to our terms of service and acknowledge you have read our privacy policy.

Not the answer you're looking for? Browse other questions tagged or ask your own question.